Find value of $|z_2+z_3|$Prove that $cos (A + B)cos (A - B) = cos ^2A - sin ^2B$Complex number modulus identity (on unit circle)Show that complex numbers are vertices of equilateral triangle$(x_n,y_n) in (x,y)mid y=kx, n=1,2,3,4Leftrightarrow$ $fracz_1-z_3z_2-z_3 div fracz_1-z_4z_2-z_4 in mathbbR, z_n = x_n+iy_n$If $z_1,z_2,z_3 in mathbbC$ and $|z_1|=|z_2|=|z_3|$ and $z_1+z_2+z_3=0$. Prove that $z_1,z_2,z_3$ are points of a equilateral triangleFor three complex numbers we have $|z_1|=1$ ,$|z_2|=2$ ,$|z_3|=3$ and $|9z​_1z_2 + 4z_1z_3 + z_2z_3|=12$Find maximum value of$ |z_1 -z_2 |^2 + |z_2 -z_3 |^2 + |z_3 -z_1 |^2 $ if $|z_1 | = 2, |z_2 | = 3, |z_3 | = 4 $The complex numbers $z_1, z_2,z_3$ are satisfying $fracz_1-z_3z_2-z_3=frac1-isqrt32$…Four complex numbers on a circle or on a straight line.How to find the maximum and minimum value of $left|(z_1-z_2)^2 + (z_2-z_3)^2 + (z_3-z_1)^2right|$ (where $|z_1|=|z_2|=|z_3|=1$)?If $fracaz_2-z_3=fracbz_3-z_1=fracc$, then why is $fraca^2z_2-z_3+fracb^2z_3-z_1+fracc^2z_1-z_2=0$?

Algorithm to convert a fixed-length string to the smallest possible collision-free representation?

How to create a hard link to an inode (ext4)?

Is there any way to damage Intellect Devourer(s) when already within a creature's skull?

Why the color red for the Republican Party

Is there an equal sign with wider gap?

Should I tell my boss the work he did was worthless

Rejected in 4th interview round citing insufficient years of experience

Finding algorithms of QGIS commands?

How do you like my writing?

How do I deal with a powergamer in a game full of beginners in a school club?

Latest web browser compatible with Windows 98

Good allowance savings plan?

Should I take out a loan for a friend to invest on my behalf?

How much attack damage does the AC boost from a shield prevent on average?

"One can do his homework in the library"

Reverse string, can I make it faster?

Why is Beresheet doing a only a one-way trip?

Good for you! in Russian

What to do when during a meeting client people start to fight (even physically) with each others?

Why is there a voltage between the mains ground and my radiator?

Why doesn't this Google Translate ad use the word "Translation" instead of "Translate"?

What is the chance of making a successful appeal to dismissal decision from a PhD program after failing the qualifying exam in the 2nd attempt?

Offered promotion but I'm leaving. Should I tell?

Is Gradient Descent central to every optimizer?



Find value of $|z_2+z_3|$


Prove that $cos (A + B)cos (A - B) = cos ^2A - sin ^2B$Complex number modulus identity (on unit circle)Show that complex numbers are vertices of equilateral triangle$(x_n,y_n) in (x,y)mid y=kx, n=1,2,3,4Leftrightarrow$ $fracz_1-z_3z_2-z_3 div fracz_1-z_4z_2-z_4 in mathbbR, z_n = x_n+iy_n$If $z_1,z_2,z_3 in mathbbC$ and $|z_1|=|z_2|=|z_3|$ and $z_1+z_2+z_3=0$. Prove that $z_1,z_2,z_3$ are points of a equilateral triangleFor three complex numbers we have $|z_1|=1$ ,$|z_2|=2$ ,$|z_3|=3$ and $|9z​_1z_2 + 4z_1z_3 + z_2z_3|=12$Find maximum value of$ |z_1 -z_2 |^2 + |z_2 -z_3 |^2 + |z_3 -z_1 |^2 $ if $|z_1 | = 2, |z_2 | = 3, |z_3 | = 4 $The complex numbers $z_1, z_2,z_3$ are satisfying $fracz_1-z_3z_2-z_3=frac1-isqrt32$…Four complex numbers on a circle or on a straight line.How to find the maximum and minimum value of $left|(z_1-z_2)^2 + (z_2-z_3)^2 + (z_3-z_1)^2right|$ (where $|z_1|=|z_2|=|z_3|=1$)?If $fraca=fracb=fracc$, then why is $fraca^2z_2-z_3+fracb^2z_3-z_1+fracc^2z_1-z_2=0$?













1












$begingroup$


Given that complex numbers $z_1,z_2,z_3$ lie on unit circle and



$$|z_1-z_2|^2+|z_1-z_3|^2=4$$ Then find value of $|z_2+z_3|$



My try:



We can take $z_1=e^ialpha$, $z_2=e^ibeta$ and $z_3=e^igamma$



So we have



$$|z_1-z_2|=2sinleft(fracalpha-beta2right)$$



$$|z_1-z_3|=2sinleft(fracalpha-gamma2right)$$



So we get:



$$sin^2left(fracalpha-beta2right)+sin^2left(fracalpha-gamma2right)=1$$



Now we have:



$$|z_2+z_3|=2cosleft(fracbeta-gamma2right)$$



Any help here?










share|cite|improve this question









$endgroup$











  • $begingroup$
    I modified the answer, taking into account a comment pointing out to me that once $z_1$ and $z_2$ are fixed, there are in fact two choices for $z_3$...
    $endgroup$
    – Matteo
    2 days ago
















1












$begingroup$


Given that complex numbers $z_1,z_2,z_3$ lie on unit circle and



$$|z_1-z_2|^2+|z_1-z_3|^2=4$$ Then find value of $|z_2+z_3|$



My try:



We can take $z_1=e^ialpha$, $z_2=e^ibeta$ and $z_3=e^igamma$



So we have



$$|z_1-z_2|=2sinleft(fracalpha-beta2right)$$



$$|z_1-z_3|=2sinleft(fracalpha-gamma2right)$$



So we get:



$$sin^2left(fracalpha-beta2right)+sin^2left(fracalpha-gamma2right)=1$$



Now we have:



$$|z_2+z_3|=2cosleft(fracbeta-gamma2right)$$



Any help here?










share|cite|improve this question









$endgroup$











  • $begingroup$
    I modified the answer, taking into account a comment pointing out to me that once $z_1$ and $z_2$ are fixed, there are in fact two choices for $z_3$...
    $endgroup$
    – Matteo
    2 days ago














1












1








1


2



$begingroup$


Given that complex numbers $z_1,z_2,z_3$ lie on unit circle and



$$|z_1-z_2|^2+|z_1-z_3|^2=4$$ Then find value of $|z_2+z_3|$



My try:



We can take $z_1=e^ialpha$, $z_2=e^ibeta$ and $z_3=e^igamma$



So we have



$$|z_1-z_2|=2sinleft(fracalpha-beta2right)$$



$$|z_1-z_3|=2sinleft(fracalpha-gamma2right)$$



So we get:



$$sin^2left(fracalpha-beta2right)+sin^2left(fracalpha-gamma2right)=1$$



Now we have:



$$|z_2+z_3|=2cosleft(fracbeta-gamma2right)$$



Any help here?










share|cite|improve this question









$endgroup$




Given that complex numbers $z_1,z_2,z_3$ lie on unit circle and



$$|z_1-z_2|^2+|z_1-z_3|^2=4$$ Then find value of $|z_2+z_3|$



My try:



We can take $z_1=e^ialpha$, $z_2=e^ibeta$ and $z_3=e^igamma$



So we have



$$|z_1-z_2|=2sinleft(fracalpha-beta2right)$$



$$|z_1-z_3|=2sinleft(fracalpha-gamma2right)$$



So we get:



$$sin^2left(fracalpha-beta2right)+sin^2left(fracalpha-gamma2right)=1$$



Now we have:



$$|z_2+z_3|=2cosleft(fracbeta-gamma2right)$$



Any help here?







algebra-precalculus trigonometry complex-numbers






share|cite|improve this question













share|cite|improve this question











share|cite|improve this question




share|cite|improve this question










asked 2 days ago









Umesh shankarUmesh shankar

2,99331220




2,99331220











  • $begingroup$
    I modified the answer, taking into account a comment pointing out to me that once $z_1$ and $z_2$ are fixed, there are in fact two choices for $z_3$...
    $endgroup$
    – Matteo
    2 days ago

















  • $begingroup$
    I modified the answer, taking into account a comment pointing out to me that once $z_1$ and $z_2$ are fixed, there are in fact two choices for $z_3$...
    $endgroup$
    – Matteo
    2 days ago
















$begingroup$
I modified the answer, taking into account a comment pointing out to me that once $z_1$ and $z_2$ are fixed, there are in fact two choices for $z_3$...
$endgroup$
– Matteo
2 days ago





$begingroup$
I modified the answer, taking into account a comment pointing out to me that once $z_1$ and $z_2$ are fixed, there are in fact two choices for $z_3$...
$endgroup$
– Matteo
2 days ago











3 Answers
3






active

oldest

votes


















2












$begingroup$

As pointed out in comment by Stancek, whom I thank, my previous answer was not correct since it only reported one solution, that was indenpendent on the choice of $z_1$, $z_2$ and $z_3$, i.e
$$|z_2+z_3|=0,$$
easily found by Pythagorean Theorem.



The general solution can be determined as follows. Use the fact that $|z|^2 = zz^*$, to re-write the equation in the form
$$|z_1|^2 - 2Rez_1z_2^* + |z_2|^2 + |z_1|^2 - 2Rez_1z_3^* + |z_3|^2=4.$$
Since the points lie on the unit circle, the equation then simplifies to
$$ Rez_1z_2^* = -Rez_1z_3^*.$$
If you set $z_1 = 1$ you get
$$Rez_2 = -Rez_3.$$
Which yields the two solutions
$$z_3 = -z_2$$
and
$$z_3 = -z_2^*.$$
The first solution leads to
$$boxedz_2+z_3,$$
that is to $z_1$, $z_2$, and $z_3$ being vertices of a right-angled triangle.
The second one, in the case $z_1=1$ can be expressed as
$$|z_2+z_3|= 2left|Imz_2right|.$$
Taking into account the arbitray phase of $z_1$ yields
$$boxed.$$
enter image description here






share|cite|improve this answer











$endgroup$








  • 1




    $begingroup$
    I think that it is not always 0. Because if you fix points $z_1$ and $z_2$, there are two choices for $z_3$. One is opposite to $z_2$, but the other one is not.
    $endgroup$
    – TStancek
    2 days ago











  • $begingroup$
    @TStancek thanks. I'll double check and correct as soon as possible
    $endgroup$
    – Matteo
    2 days ago










  • $begingroup$
    @TStancek check out the changes if you like. Now it should be correct, I hope. Thanks for the correction!
    $endgroup$
    – Matteo
    2 days ago






  • 1




    $begingroup$
    To be honest, I am not sure. I think you should probably multiply all $z_1,z_2$ and $z_3$ by $z_1^-1$ to move the starting position of points to that special position where $z_1 = 1$, than do the complex conjugation of opposite point and only than multiply it back by $z_1$. So actually I think the second $z_3$ should be equal to $overline-(z_2cdot z_1^-1) cdot z_1$. But I am really not sure and currently dont have time to verify this.
    $endgroup$
    – TStancek
    yesterday


















2












$begingroup$

Note that if $|z|=1$, then $overlinez=1/z$. Thus for $z_1$, $z_2$, and $z_3$ on the unit circle, we have



$$beginalign
|z_1-z_2|^2+|z_1-z_3|^2
&=left(z_1-z_2right)left(1over z_1-1over z_2right)+left(z_1-z_3right)left(1over z_1-1over z_3right)\
&=4-left(z_1over z_2+z_2over z_1+z_1over z_3+z_3over z_1 right)\
&=4-left((z_1^2+z_2z_3)(z_2+z_3)over z_1z_2z_3 right)
endalign$$



It follows that $|z_1-z_2|^2+|z_1-z_3|^2=4$ if and only if $(z_1^2+z_2z_3)(z_2+z_3)=0$. Since we can always arrange for $z_1^2=-z_2z_3$ on the unit circle, $|z_2+z_3|$ can take any value between $0$ and $2$.






share|cite|improve this answer









$endgroup$




















    1












    $begingroup$

    Hint: Writing $$fracbeta-gamma2=fracbeta-alpha2+fracalpha-gamma2$$ and combing this with trigonometric identity for the sum of cosine $$cos(x+y)=cos(x)cos(y)-sin(x)sin(y)$$ we get
    beginaligncosleft(fracbeta-gamma2right)&=cosleft(fracbeta-alpha2+fracalpha-gamma2right)\
    &=cosleft(fracbeta-alpha2right)cdotcosleft(fracalpha-gamma2right)-sinleft(fracbeta-alpha2right)cdotsinleft(fracalpha-gamma2right).endalign

    Now rewrite the identity
    $$sin^2left(fracalpha-beta2right)+sin^2left(fracalpha-gamma2right)=1$$
    as
    $$sin^2left(fracalpha-beta2right)=1-sin^2left(fracalpha-gamma2right)=cos^2left(fracalpha-gamma2right),$$
    and plug this into the previous to conclude.






    share|cite|improve this answer











    $endgroup$












    • $begingroup$
      Using math.stackexchange.com/questions/345703/…, $$0=cos^2dfracalpha-gamma2-sin^2dfracalpha-beta2=cosdfracbeta-gamma2cosdfrac2alpha-beta-gamma2$$
      $endgroup$
      – lab bhattacharjee
      2 days ago










    Your Answer





    StackExchange.ifUsing("editor", function ()
    return StackExchange.using("mathjaxEditing", function ()
    StackExchange.MarkdownEditor.creationCallbacks.add(function (editor, postfix)
    StackExchange.mathjaxEditing.prepareWmdForMathJax(editor, postfix, [["$", "$"], ["\\(","\\)"]]);
    );
    );
    , "mathjax-editing");

    StackExchange.ready(function()
    var channelOptions =
    tags: "".split(" "),
    id: "69"
    ;
    initTagRenderer("".split(" "), "".split(" "), channelOptions);

    StackExchange.using("externalEditor", function()
    // Have to fire editor after snippets, if snippets enabled
    if (StackExchange.settings.snippets.snippetsEnabled)
    StackExchange.using("snippets", function()
    createEditor();
    );

    else
    createEditor();

    );

    function createEditor()
    StackExchange.prepareEditor(
    heartbeatType: 'answer',
    autoActivateHeartbeat: false,
    convertImagesToLinks: true,
    noModals: true,
    showLowRepImageUploadWarning: true,
    reputationToPostImages: 10,
    bindNavPrevention: true,
    postfix: "",
    imageUploader:
    brandingHtml: "Powered by u003ca class="icon-imgur-white" href="https://imgur.com/"u003eu003c/au003e",
    contentPolicyHtml: "User contributions licensed under u003ca href="https://creativecommons.org/licenses/by-sa/3.0/"u003ecc by-sa 3.0 with attribution requiredu003c/au003e u003ca href="https://stackoverflow.com/legal/content-policy"u003e(content policy)u003c/au003e",
    allowUrls: true
    ,
    noCode: true, onDemand: true,
    discardSelector: ".discard-answer"
    ,immediatelyShowMarkdownHelp:true
    );



    );













    draft saved

    draft discarded


















    StackExchange.ready(
    function ()
    StackExchange.openid.initPostLogin('.new-post-login', 'https%3a%2f%2fmath.stackexchange.com%2fquestions%2f3142187%2ffind-value-of-z-2z-3%23new-answer', 'question_page');

    );

    Post as a guest















    Required, but never shown

























    3 Answers
    3






    active

    oldest

    votes








    3 Answers
    3






    active

    oldest

    votes









    active

    oldest

    votes






    active

    oldest

    votes









    2












    $begingroup$

    As pointed out in comment by Stancek, whom I thank, my previous answer was not correct since it only reported one solution, that was indenpendent on the choice of $z_1$, $z_2$ and $z_3$, i.e
    $$|z_2+z_3|=0,$$
    easily found by Pythagorean Theorem.



    The general solution can be determined as follows. Use the fact that $|z|^2 = zz^*$, to re-write the equation in the form
    $$|z_1|^2 - 2Rez_1z_2^* + |z_2|^2 + |z_1|^2 - 2Rez_1z_3^* + |z_3|^2=4.$$
    Since the points lie on the unit circle, the equation then simplifies to
    $$ Rez_1z_2^* = -Rez_1z_3^*.$$
    If you set $z_1 = 1$ you get
    $$Rez_2 = -Rez_3.$$
    Which yields the two solutions
    $$z_3 = -z_2$$
    and
    $$z_3 = -z_2^*.$$
    The first solution leads to
    $$boxedz_2+z_3,$$
    that is to $z_1$, $z_2$, and $z_3$ being vertices of a right-angled triangle.
    The second one, in the case $z_1=1$ can be expressed as
    $$|z_2+z_3|= 2left|Imz_2right|.$$
    Taking into account the arbitray phase of $z_1$ yields
    $$boxed.$$
    enter image description here






    share|cite|improve this answer











    $endgroup$








    • 1




      $begingroup$
      I think that it is not always 0. Because if you fix points $z_1$ and $z_2$, there are two choices for $z_3$. One is opposite to $z_2$, but the other one is not.
      $endgroup$
      – TStancek
      2 days ago











    • $begingroup$
      @TStancek thanks. I'll double check and correct as soon as possible
      $endgroup$
      – Matteo
      2 days ago










    • $begingroup$
      @TStancek check out the changes if you like. Now it should be correct, I hope. Thanks for the correction!
      $endgroup$
      – Matteo
      2 days ago






    • 1




      $begingroup$
      To be honest, I am not sure. I think you should probably multiply all $z_1,z_2$ and $z_3$ by $z_1^-1$ to move the starting position of points to that special position where $z_1 = 1$, than do the complex conjugation of opposite point and only than multiply it back by $z_1$. So actually I think the second $z_3$ should be equal to $overline-(z_2cdot z_1^-1) cdot z_1$. But I am really not sure and currently dont have time to verify this.
      $endgroup$
      – TStancek
      yesterday















    2












    $begingroup$

    As pointed out in comment by Stancek, whom I thank, my previous answer was not correct since it only reported one solution, that was indenpendent on the choice of $z_1$, $z_2$ and $z_3$, i.e
    $$|z_2+z_3|=0,$$
    easily found by Pythagorean Theorem.



    The general solution can be determined as follows. Use the fact that $|z|^2 = zz^*$, to re-write the equation in the form
    $$|z_1|^2 - 2Rez_1z_2^* + |z_2|^2 + |z_1|^2 - 2Rez_1z_3^* + |z_3|^2=4.$$
    Since the points lie on the unit circle, the equation then simplifies to
    $$ Rez_1z_2^* = -Rez_1z_3^*.$$
    If you set $z_1 = 1$ you get
    $$Rez_2 = -Rez_3.$$
    Which yields the two solutions
    $$z_3 = -z_2$$
    and
    $$z_3 = -z_2^*.$$
    The first solution leads to
    $$boxedz_2+z_3,$$
    that is to $z_1$, $z_2$, and $z_3$ being vertices of a right-angled triangle.
    The second one, in the case $z_1=1$ can be expressed as
    $$|z_2+z_3|= 2left|Imz_2right|.$$
    Taking into account the arbitray phase of $z_1$ yields
    $$boxed.$$
    enter image description here






    share|cite|improve this answer











    $endgroup$








    • 1




      $begingroup$
      I think that it is not always 0. Because if you fix points $z_1$ and $z_2$, there are two choices for $z_3$. One is opposite to $z_2$, but the other one is not.
      $endgroup$
      – TStancek
      2 days ago











    • $begingroup$
      @TStancek thanks. I'll double check and correct as soon as possible
      $endgroup$
      – Matteo
      2 days ago










    • $begingroup$
      @TStancek check out the changes if you like. Now it should be correct, I hope. Thanks for the correction!
      $endgroup$
      – Matteo
      2 days ago






    • 1




      $begingroup$
      To be honest, I am not sure. I think you should probably multiply all $z_1,z_2$ and $z_3$ by $z_1^-1$ to move the starting position of points to that special position where $z_1 = 1$, than do the complex conjugation of opposite point and only than multiply it back by $z_1$. So actually I think the second $z_3$ should be equal to $overline-(z_2cdot z_1^-1) cdot z_1$. But I am really not sure and currently dont have time to verify this.
      $endgroup$
      – TStancek
      yesterday













    2












    2








    2





    $begingroup$

    As pointed out in comment by Stancek, whom I thank, my previous answer was not correct since it only reported one solution, that was indenpendent on the choice of $z_1$, $z_2$ and $z_3$, i.e
    $$|z_2+z_3|=0,$$
    easily found by Pythagorean Theorem.



    The general solution can be determined as follows. Use the fact that $|z|^2 = zz^*$, to re-write the equation in the form
    $$|z_1|^2 - 2Rez_1z_2^* + |z_2|^2 + |z_1|^2 - 2Rez_1z_3^* + |z_3|^2=4.$$
    Since the points lie on the unit circle, the equation then simplifies to
    $$ Rez_1z_2^* = -Rez_1z_3^*.$$
    If you set $z_1 = 1$ you get
    $$Rez_2 = -Rez_3.$$
    Which yields the two solutions
    $$z_3 = -z_2$$
    and
    $$z_3 = -z_2^*.$$
    The first solution leads to
    $$boxedz_2+z_3,$$
    that is to $z_1$, $z_2$, and $z_3$ being vertices of a right-angled triangle.
    The second one, in the case $z_1=1$ can be expressed as
    $$|z_2+z_3|= 2left|Imz_2right|.$$
    Taking into account the arbitray phase of $z_1$ yields
    $$boxed.$$
    enter image description here






    share|cite|improve this answer











    $endgroup$



    As pointed out in comment by Stancek, whom I thank, my previous answer was not correct since it only reported one solution, that was indenpendent on the choice of $z_1$, $z_2$ and $z_3$, i.e
    $$|z_2+z_3|=0,$$
    easily found by Pythagorean Theorem.



    The general solution can be determined as follows. Use the fact that $|z|^2 = zz^*$, to re-write the equation in the form
    $$|z_1|^2 - 2Rez_1z_2^* + |z_2|^2 + |z_1|^2 - 2Rez_1z_3^* + |z_3|^2=4.$$
    Since the points lie on the unit circle, the equation then simplifies to
    $$ Rez_1z_2^* = -Rez_1z_3^*.$$
    If you set $z_1 = 1$ you get
    $$Rez_2 = -Rez_3.$$
    Which yields the two solutions
    $$z_3 = -z_2$$
    and
    $$z_3 = -z_2^*.$$
    The first solution leads to
    $$boxedz_2+z_3,$$
    that is to $z_1$, $z_2$, and $z_3$ being vertices of a right-angled triangle.
    The second one, in the case $z_1=1$ can be expressed as
    $$|z_2+z_3|= 2left|Imz_2right|.$$
    Taking into account the arbitray phase of $z_1$ yields
    $$boxed.$$
    enter image description here







    share|cite|improve this answer














    share|cite|improve this answer



    share|cite|improve this answer








    edited 2 days ago

























    answered 2 days ago









    MatteoMatteo

    1,032313




    1,032313







    • 1




      $begingroup$
      I think that it is not always 0. Because if you fix points $z_1$ and $z_2$, there are two choices for $z_3$. One is opposite to $z_2$, but the other one is not.
      $endgroup$
      – TStancek
      2 days ago











    • $begingroup$
      @TStancek thanks. I'll double check and correct as soon as possible
      $endgroup$
      – Matteo
      2 days ago










    • $begingroup$
      @TStancek check out the changes if you like. Now it should be correct, I hope. Thanks for the correction!
      $endgroup$
      – Matteo
      2 days ago






    • 1




      $begingroup$
      To be honest, I am not sure. I think you should probably multiply all $z_1,z_2$ and $z_3$ by $z_1^-1$ to move the starting position of points to that special position where $z_1 = 1$, than do the complex conjugation of opposite point and only than multiply it back by $z_1$. So actually I think the second $z_3$ should be equal to $overline-(z_2cdot z_1^-1) cdot z_1$. But I am really not sure and currently dont have time to verify this.
      $endgroup$
      – TStancek
      yesterday












    • 1




      $begingroup$
      I think that it is not always 0. Because if you fix points $z_1$ and $z_2$, there are two choices for $z_3$. One is opposite to $z_2$, but the other one is not.
      $endgroup$
      – TStancek
      2 days ago











    • $begingroup$
      @TStancek thanks. I'll double check and correct as soon as possible
      $endgroup$
      – Matteo
      2 days ago










    • $begingroup$
      @TStancek check out the changes if you like. Now it should be correct, I hope. Thanks for the correction!
      $endgroup$
      – Matteo
      2 days ago






    • 1




      $begingroup$
      To be honest, I am not sure. I think you should probably multiply all $z_1,z_2$ and $z_3$ by $z_1^-1$ to move the starting position of points to that special position where $z_1 = 1$, than do the complex conjugation of opposite point and only than multiply it back by $z_1$. So actually I think the second $z_3$ should be equal to $overline-(z_2cdot z_1^-1) cdot z_1$. But I am really not sure and currently dont have time to verify this.
      $endgroup$
      – TStancek
      yesterday







    1




    1




    $begingroup$
    I think that it is not always 0. Because if you fix points $z_1$ and $z_2$, there are two choices for $z_3$. One is opposite to $z_2$, but the other one is not.
    $endgroup$
    – TStancek
    2 days ago





    $begingroup$
    I think that it is not always 0. Because if you fix points $z_1$ and $z_2$, there are two choices for $z_3$. One is opposite to $z_2$, but the other one is not.
    $endgroup$
    – TStancek
    2 days ago













    $begingroup$
    @TStancek thanks. I'll double check and correct as soon as possible
    $endgroup$
    – Matteo
    2 days ago




    $begingroup$
    @TStancek thanks. I'll double check and correct as soon as possible
    $endgroup$
    – Matteo
    2 days ago












    $begingroup$
    @TStancek check out the changes if you like. Now it should be correct, I hope. Thanks for the correction!
    $endgroup$
    – Matteo
    2 days ago




    $begingroup$
    @TStancek check out the changes if you like. Now it should be correct, I hope. Thanks for the correction!
    $endgroup$
    – Matteo
    2 days ago




    1




    1




    $begingroup$
    To be honest, I am not sure. I think you should probably multiply all $z_1,z_2$ and $z_3$ by $z_1^-1$ to move the starting position of points to that special position where $z_1 = 1$, than do the complex conjugation of opposite point and only than multiply it back by $z_1$. So actually I think the second $z_3$ should be equal to $overline-(z_2cdot z_1^-1) cdot z_1$. But I am really not sure and currently dont have time to verify this.
    $endgroup$
    – TStancek
    yesterday




    $begingroup$
    To be honest, I am not sure. I think you should probably multiply all $z_1,z_2$ and $z_3$ by $z_1^-1$ to move the starting position of points to that special position where $z_1 = 1$, than do the complex conjugation of opposite point and only than multiply it back by $z_1$. So actually I think the second $z_3$ should be equal to $overline-(z_2cdot z_1^-1) cdot z_1$. But I am really not sure and currently dont have time to verify this.
    $endgroup$
    – TStancek
    yesterday











    2












    $begingroup$

    Note that if $|z|=1$, then $overlinez=1/z$. Thus for $z_1$, $z_2$, and $z_3$ on the unit circle, we have



    $$beginalign
    |z_1-z_2|^2+|z_1-z_3|^2
    &=left(z_1-z_2right)left(1over z_1-1over z_2right)+left(z_1-z_3right)left(1over z_1-1over z_3right)\
    &=4-left(z_1over z_2+z_2over z_1+z_1over z_3+z_3over z_1 right)\
    &=4-left((z_1^2+z_2z_3)(z_2+z_3)over z_1z_2z_3 right)
    endalign$$



    It follows that $|z_1-z_2|^2+|z_1-z_3|^2=4$ if and only if $(z_1^2+z_2z_3)(z_2+z_3)=0$. Since we can always arrange for $z_1^2=-z_2z_3$ on the unit circle, $|z_2+z_3|$ can take any value between $0$ and $2$.






    share|cite|improve this answer









    $endgroup$

















      2












      $begingroup$

      Note that if $|z|=1$, then $overlinez=1/z$. Thus for $z_1$, $z_2$, and $z_3$ on the unit circle, we have



      $$beginalign
      |z_1-z_2|^2+|z_1-z_3|^2
      &=left(z_1-z_2right)left(1over z_1-1over z_2right)+left(z_1-z_3right)left(1over z_1-1over z_3right)\
      &=4-left(z_1over z_2+z_2over z_1+z_1over z_3+z_3over z_1 right)\
      &=4-left((z_1^2+z_2z_3)(z_2+z_3)over z_1z_2z_3 right)
      endalign$$



      It follows that $|z_1-z_2|^2+|z_1-z_3|^2=4$ if and only if $(z_1^2+z_2z_3)(z_2+z_3)=0$. Since we can always arrange for $z_1^2=-z_2z_3$ on the unit circle, $|z_2+z_3|$ can take any value between $0$ and $2$.






      share|cite|improve this answer









      $endgroup$















        2












        2








        2





        $begingroup$

        Note that if $|z|=1$, then $overlinez=1/z$. Thus for $z_1$, $z_2$, and $z_3$ on the unit circle, we have



        $$beginalign
        |z_1-z_2|^2+|z_1-z_3|^2
        &=left(z_1-z_2right)left(1over z_1-1over z_2right)+left(z_1-z_3right)left(1over z_1-1over z_3right)\
        &=4-left(z_1over z_2+z_2over z_1+z_1over z_3+z_3over z_1 right)\
        &=4-left((z_1^2+z_2z_3)(z_2+z_3)over z_1z_2z_3 right)
        endalign$$



        It follows that $|z_1-z_2|^2+|z_1-z_3|^2=4$ if and only if $(z_1^2+z_2z_3)(z_2+z_3)=0$. Since we can always arrange for $z_1^2=-z_2z_3$ on the unit circle, $|z_2+z_3|$ can take any value between $0$ and $2$.






        share|cite|improve this answer









        $endgroup$



        Note that if $|z|=1$, then $overlinez=1/z$. Thus for $z_1$, $z_2$, and $z_3$ on the unit circle, we have



        $$beginalign
        |z_1-z_2|^2+|z_1-z_3|^2
        &=left(z_1-z_2right)left(1over z_1-1over z_2right)+left(z_1-z_3right)left(1over z_1-1over z_3right)\
        &=4-left(z_1over z_2+z_2over z_1+z_1over z_3+z_3over z_1 right)\
        &=4-left((z_1^2+z_2z_3)(z_2+z_3)over z_1z_2z_3 right)
        endalign$$



        It follows that $|z_1-z_2|^2+|z_1-z_3|^2=4$ if and only if $(z_1^2+z_2z_3)(z_2+z_3)=0$. Since we can always arrange for $z_1^2=-z_2z_3$ on the unit circle, $|z_2+z_3|$ can take any value between $0$ and $2$.







        share|cite|improve this answer












        share|cite|improve this answer



        share|cite|improve this answer










        answered 2 days ago









        Barry CipraBarry Cipra

        60k654126




        60k654126





















            1












            $begingroup$

            Hint: Writing $$fracbeta-gamma2=fracbeta-alpha2+fracalpha-gamma2$$ and combing this with trigonometric identity for the sum of cosine $$cos(x+y)=cos(x)cos(y)-sin(x)sin(y)$$ we get
            beginaligncosleft(fracbeta-gamma2right)&=cosleft(fracbeta-alpha2+fracalpha-gamma2right)\
            &=cosleft(fracbeta-alpha2right)cdotcosleft(fracalpha-gamma2right)-sinleft(fracbeta-alpha2right)cdotsinleft(fracalpha-gamma2right).endalign

            Now rewrite the identity
            $$sin^2left(fracalpha-beta2right)+sin^2left(fracalpha-gamma2right)=1$$
            as
            $$sin^2left(fracalpha-beta2right)=1-sin^2left(fracalpha-gamma2right)=cos^2left(fracalpha-gamma2right),$$
            and plug this into the previous to conclude.






            share|cite|improve this answer











            $endgroup$












            • $begingroup$
              Using math.stackexchange.com/questions/345703/…, $$0=cos^2dfracalpha-gamma2-sin^2dfracalpha-beta2=cosdfracbeta-gamma2cosdfrac2alpha-beta-gamma2$$
              $endgroup$
              – lab bhattacharjee
              2 days ago















            1












            $begingroup$

            Hint: Writing $$fracbeta-gamma2=fracbeta-alpha2+fracalpha-gamma2$$ and combing this with trigonometric identity for the sum of cosine $$cos(x+y)=cos(x)cos(y)-sin(x)sin(y)$$ we get
            beginaligncosleft(fracbeta-gamma2right)&=cosleft(fracbeta-alpha2+fracalpha-gamma2right)\
            &=cosleft(fracbeta-alpha2right)cdotcosleft(fracalpha-gamma2right)-sinleft(fracbeta-alpha2right)cdotsinleft(fracalpha-gamma2right).endalign

            Now rewrite the identity
            $$sin^2left(fracalpha-beta2right)+sin^2left(fracalpha-gamma2right)=1$$
            as
            $$sin^2left(fracalpha-beta2right)=1-sin^2left(fracalpha-gamma2right)=cos^2left(fracalpha-gamma2right),$$
            and plug this into the previous to conclude.






            share|cite|improve this answer











            $endgroup$












            • $begingroup$
              Using math.stackexchange.com/questions/345703/…, $$0=cos^2dfracalpha-gamma2-sin^2dfracalpha-beta2=cosdfracbeta-gamma2cosdfrac2alpha-beta-gamma2$$
              $endgroup$
              – lab bhattacharjee
              2 days ago













            1












            1








            1





            $begingroup$

            Hint: Writing $$fracbeta-gamma2=fracbeta-alpha2+fracalpha-gamma2$$ and combing this with trigonometric identity for the sum of cosine $$cos(x+y)=cos(x)cos(y)-sin(x)sin(y)$$ we get
            beginaligncosleft(fracbeta-gamma2right)&=cosleft(fracbeta-alpha2+fracalpha-gamma2right)\
            &=cosleft(fracbeta-alpha2right)cdotcosleft(fracalpha-gamma2right)-sinleft(fracbeta-alpha2right)cdotsinleft(fracalpha-gamma2right).endalign

            Now rewrite the identity
            $$sin^2left(fracalpha-beta2right)+sin^2left(fracalpha-gamma2right)=1$$
            as
            $$sin^2left(fracalpha-beta2right)=1-sin^2left(fracalpha-gamma2right)=cos^2left(fracalpha-gamma2right),$$
            and plug this into the previous to conclude.






            share|cite|improve this answer











            $endgroup$



            Hint: Writing $$fracbeta-gamma2=fracbeta-alpha2+fracalpha-gamma2$$ and combing this with trigonometric identity for the sum of cosine $$cos(x+y)=cos(x)cos(y)-sin(x)sin(y)$$ we get
            beginaligncosleft(fracbeta-gamma2right)&=cosleft(fracbeta-alpha2+fracalpha-gamma2right)\
            &=cosleft(fracbeta-alpha2right)cdotcosleft(fracalpha-gamma2right)-sinleft(fracbeta-alpha2right)cdotsinleft(fracalpha-gamma2right).endalign

            Now rewrite the identity
            $$sin^2left(fracalpha-beta2right)+sin^2left(fracalpha-gamma2right)=1$$
            as
            $$sin^2left(fracalpha-beta2right)=1-sin^2left(fracalpha-gamma2right)=cos^2left(fracalpha-gamma2right),$$
            and plug this into the previous to conclude.







            share|cite|improve this answer














            share|cite|improve this answer



            share|cite|improve this answer








            edited 2 days ago

























            answered 2 days ago









            b00n heTb00n heT

            10.5k12235




            10.5k12235











            • $begingroup$
              Using math.stackexchange.com/questions/345703/…, $$0=cos^2dfracalpha-gamma2-sin^2dfracalpha-beta2=cosdfracbeta-gamma2cosdfrac2alpha-beta-gamma2$$
              $endgroup$
              – lab bhattacharjee
              2 days ago
















            • $begingroup$
              Using math.stackexchange.com/questions/345703/…, $$0=cos^2dfracalpha-gamma2-sin^2dfracalpha-beta2=cosdfracbeta-gamma2cosdfrac2alpha-beta-gamma2$$
              $endgroup$
              – lab bhattacharjee
              2 days ago















            $begingroup$
            Using math.stackexchange.com/questions/345703/…, $$0=cos^2dfracalpha-gamma2-sin^2dfracalpha-beta2=cosdfracbeta-gamma2cosdfrac2alpha-beta-gamma2$$
            $endgroup$
            – lab bhattacharjee
            2 days ago




            $begingroup$
            Using math.stackexchange.com/questions/345703/…, $$0=cos^2dfracalpha-gamma2-sin^2dfracalpha-beta2=cosdfracbeta-gamma2cosdfrac2alpha-beta-gamma2$$
            $endgroup$
            – lab bhattacharjee
            2 days ago

















            draft saved

            draft discarded
















































            Thanks for contributing an answer to Mathematics Stack Exchange!


            • Please be sure to answer the question. Provide details and share your research!

            But avoid


            • Asking for help, clarification, or responding to other answers.

            • Making statements based on opinion; back them up with references or personal experience.

            Use MathJax to format equations. MathJax reference.


            To learn more, see our tips on writing great answers.




            draft saved


            draft discarded














            StackExchange.ready(
            function ()
            StackExchange.openid.initPostLogin('.new-post-login', 'https%3a%2f%2fmath.stackexchange.com%2fquestions%2f3142187%2ffind-value-of-z-2z-3%23new-answer', 'question_page');

            );

            Post as a guest















            Required, but never shown





















































            Required, but never shown














            Required, but never shown












            Required, but never shown







            Required, but never shown

































            Required, but never shown














            Required, but never shown












            Required, but never shown







            Required, but never shown







            Popular posts from this blog

            Lowndes Grove History Architecture References Navigation menu32°48′6″N 79°57′58″W / 32.80167°N 79.96611°W / 32.80167; -79.9661132°48′6″N 79°57′58″W / 32.80167°N 79.96611°W / 32.80167; -79.9661178002500"National Register Information System"Historic houses of South Carolina"Lowndes Grove""+32° 48' 6.00", −79° 57' 58.00""Lowndes Grove, Charleston County (260 St. Margaret St., Charleston)""Lowndes Grove"The Charleston ExpositionIt Happened in South Carolina"Lowndes Grove (House), Saint Margaret Street & Sixth Avenue, Charleston, Charleston County, SC(Photographs)"Plantations of the Carolina Low Countrye

            random experiment with two different functions on unit interval Announcing the arrival of Valued Associate #679: Cesar Manara Planned maintenance scheduled April 23, 2019 at 00:00UTC (8:00pm US/Eastern)Random variable and probability space notionsRandom Walk with EdgesFinding functions where the increase over a random interval is Poisson distributedNumber of days until dayCan an observed event in fact be of zero probability?Unit random processmodels of coins and uniform distributionHow to get the number of successes given $n$ trials , probability $P$ and a random variable $X$Absorbing Markov chain in a computer. Is “almost every” turned into always convergence in computer executions?Stopped random walk is not uniformly integrable

            How should I support this large drywall patch? Planned maintenance scheduled April 23, 2019 at 00:00UTC (8:00pm US/Eastern) Announcing the arrival of Valued Associate #679: Cesar Manara Unicorn Meta Zoo #1: Why another podcast?How do I cover large gaps in drywall?How do I keep drywall around a patch from crumbling?Can I glue a second layer of drywall?How to patch long strip on drywall?Large drywall patch: how to avoid bulging seams?Drywall Mesh Patch vs. Bulge? To remove or not to remove?How to fix this drywall job?Prep drywall before backsplashWhat's the best way to fix this horrible drywall patch job?Drywall patching using 3M Patch Plus Primer